Proving $binom{n+1+m}{n+1}=sum_{k=0}^m(k+1)binom{n+m-k}{n}$












2












$begingroup$



Use any method to prove that
$$binom{n+1+m}{n+1}=sum_{k=0}^m(k+1)binom{n+m-k}{n}$$




My Try:



Base case: Let $m=1$



LHS$$binom{n+1+m}{n+1}=binom{n+2}{n+1}=(n+2)$$
RHS$$sum_{k=0}^m(k+1)binom{n+m-k}{n}=binom{n+1-0}{n}+(1+1)binom{n+1-1}{n}$$
$$=frac{(n+1)!}{n!}+2$$
$$=(n+3)$$



If $m=2$



LHS$$binom{n+3}{n+1}=frac{n^2+5n+6}{2!}$$
RHS$$=binom{n+2}{n}+2binom{n+1}{n}+3binom{n}{n}$$
$$=frac{n^2+3n+2+4n+4+6}{2}=frac{n^2+7n+12}{2}$$



Clearly $LHSne RHS$



If LHS and RHS are not equal then how to prove this proof? Can anyone explain how to prove this.










share|cite|improve this question









$endgroup$








  • 4




    $begingroup$
    If something is not true, you cannot prove that it is true.
    $endgroup$
    – Batominovski
    Dec 8 '18 at 16:46










  • $begingroup$
    @Batominovski If it is wrong, then why am I asked to prove the equation.
    $endgroup$
    – user982787
    Dec 8 '18 at 17:23






  • 1




    $begingroup$
    Mistakes happen. I hope you understand that humans are not perfect, regardless of their intelligence and experiences.
    $endgroup$
    – Batominovski
    Dec 8 '18 at 17:26


















2












$begingroup$



Use any method to prove that
$$binom{n+1+m}{n+1}=sum_{k=0}^m(k+1)binom{n+m-k}{n}$$




My Try:



Base case: Let $m=1$



LHS$$binom{n+1+m}{n+1}=binom{n+2}{n+1}=(n+2)$$
RHS$$sum_{k=0}^m(k+1)binom{n+m-k}{n}=binom{n+1-0}{n}+(1+1)binom{n+1-1}{n}$$
$$=frac{(n+1)!}{n!}+2$$
$$=(n+3)$$



If $m=2$



LHS$$binom{n+3}{n+1}=frac{n^2+5n+6}{2!}$$
RHS$$=binom{n+2}{n}+2binom{n+1}{n}+3binom{n}{n}$$
$$=frac{n^2+3n+2+4n+4+6}{2}=frac{n^2+7n+12}{2}$$



Clearly $LHSne RHS$



If LHS and RHS are not equal then how to prove this proof? Can anyone explain how to prove this.










share|cite|improve this question









$endgroup$








  • 4




    $begingroup$
    If something is not true, you cannot prove that it is true.
    $endgroup$
    – Batominovski
    Dec 8 '18 at 16:46










  • $begingroup$
    @Batominovski If it is wrong, then why am I asked to prove the equation.
    $endgroup$
    – user982787
    Dec 8 '18 at 17:23






  • 1




    $begingroup$
    Mistakes happen. I hope you understand that humans are not perfect, regardless of their intelligence and experiences.
    $endgroup$
    – Batominovski
    Dec 8 '18 at 17:26
















2












2








2





$begingroup$



Use any method to prove that
$$binom{n+1+m}{n+1}=sum_{k=0}^m(k+1)binom{n+m-k}{n}$$




My Try:



Base case: Let $m=1$



LHS$$binom{n+1+m}{n+1}=binom{n+2}{n+1}=(n+2)$$
RHS$$sum_{k=0}^m(k+1)binom{n+m-k}{n}=binom{n+1-0}{n}+(1+1)binom{n+1-1}{n}$$
$$=frac{(n+1)!}{n!}+2$$
$$=(n+3)$$



If $m=2$



LHS$$binom{n+3}{n+1}=frac{n^2+5n+6}{2!}$$
RHS$$=binom{n+2}{n}+2binom{n+1}{n}+3binom{n}{n}$$
$$=frac{n^2+3n+2+4n+4+6}{2}=frac{n^2+7n+12}{2}$$



Clearly $LHSne RHS$



If LHS and RHS are not equal then how to prove this proof? Can anyone explain how to prove this.










share|cite|improve this question









$endgroup$





Use any method to prove that
$$binom{n+1+m}{n+1}=sum_{k=0}^m(k+1)binom{n+m-k}{n}$$




My Try:



Base case: Let $m=1$



LHS$$binom{n+1+m}{n+1}=binom{n+2}{n+1}=(n+2)$$
RHS$$sum_{k=0}^m(k+1)binom{n+m-k}{n}=binom{n+1-0}{n}+(1+1)binom{n+1-1}{n}$$
$$=frac{(n+1)!}{n!}+2$$
$$=(n+3)$$



If $m=2$



LHS$$binom{n+3}{n+1}=frac{n^2+5n+6}{2!}$$
RHS$$=binom{n+2}{n}+2binom{n+1}{n}+3binom{n}{n}$$
$$=frac{n^2+3n+2+4n+4+6}{2}=frac{n^2+7n+12}{2}$$



Clearly $LHSne RHS$



If LHS and RHS are not equal then how to prove this proof? Can anyone explain how to prove this.







combinatorics discrete-mathematics induction combinatorial-proofs






share|cite|improve this question













share|cite|improve this question











share|cite|improve this question




share|cite|improve this question










asked Dec 8 '18 at 16:37









user982787user982787

1117




1117








  • 4




    $begingroup$
    If something is not true, you cannot prove that it is true.
    $endgroup$
    – Batominovski
    Dec 8 '18 at 16:46










  • $begingroup$
    @Batominovski If it is wrong, then why am I asked to prove the equation.
    $endgroup$
    – user982787
    Dec 8 '18 at 17:23






  • 1




    $begingroup$
    Mistakes happen. I hope you understand that humans are not perfect, regardless of their intelligence and experiences.
    $endgroup$
    – Batominovski
    Dec 8 '18 at 17:26
















  • 4




    $begingroup$
    If something is not true, you cannot prove that it is true.
    $endgroup$
    – Batominovski
    Dec 8 '18 at 16:46










  • $begingroup$
    @Batominovski If it is wrong, then why am I asked to prove the equation.
    $endgroup$
    – user982787
    Dec 8 '18 at 17:23






  • 1




    $begingroup$
    Mistakes happen. I hope you understand that humans are not perfect, regardless of their intelligence and experiences.
    $endgroup$
    – Batominovski
    Dec 8 '18 at 17:26










4




4




$begingroup$
If something is not true, you cannot prove that it is true.
$endgroup$
– Batominovski
Dec 8 '18 at 16:46




$begingroup$
If something is not true, you cannot prove that it is true.
$endgroup$
– Batominovski
Dec 8 '18 at 16:46












$begingroup$
@Batominovski If it is wrong, then why am I asked to prove the equation.
$endgroup$
– user982787
Dec 8 '18 at 17:23




$begingroup$
@Batominovski If it is wrong, then why am I asked to prove the equation.
$endgroup$
– user982787
Dec 8 '18 at 17:23




1




1




$begingroup$
Mistakes happen. I hope you understand that humans are not perfect, regardless of their intelligence and experiences.
$endgroup$
– Batominovski
Dec 8 '18 at 17:26






$begingroup$
Mistakes happen. I hope you understand that humans are not perfect, regardless of their intelligence and experiences.
$endgroup$
– Batominovski
Dec 8 '18 at 17:26












2 Answers
2






active

oldest

votes


















3












$begingroup$

$newcommand{bbx}[1]{,bbox[15px,border:1px groove navy]{displaystyle{#1}},}
newcommand{braces}[1]{leftlbrace,{#1},rightrbrace}
newcommand{bracks}[1]{leftlbrack,{#1},rightrbrack}
newcommand{dd}{mathrm{d}}
newcommand{ds}[1]{displaystyle{#1}}
newcommand{expo}[1]{,mathrm{e}^{#1},}
newcommand{ic}{mathrm{i}}
newcommand{mc}[1]{mathcal{#1}}
newcommand{mrm}[1]{mathrm{#1}}
newcommand{pars}[1]{left(,{#1},right)}
newcommand{partiald}[3]{frac{partial^{#1} #2}{partial #3^{#1}}}
newcommand{root}[2]{,sqrt[#1]{,{#2},},}
newcommand{totald}[3]{frac{mathrm{d}^{#1} #2}{mathrm{d} #3^{#1}}}
newcommand{verts}[1]{leftvert,{#1},rightvert}$




$ds{require{cancel}bcancel{cancel{n + 1 + m choose n + 1}} =
sum_{k = 0}^{m}pars{k + 1}{n + m - k choose n}: {LARGE ?}}$
.


The right answer is
$bbx{ds{n + m + 2 choose n + 2}}$




$$
bbx{mbox{Note that} {n + m - k choose n} = 0
mbox{when} k > m}
$$




begin{align}
&bbox[10px,#ffd]{sum_{k = 0}^{m}pars{k + 1}
{n + m - k choose n}} =
sum_{k = 0}^{infty}pars{k + 1}{n + m - k choose m - k}
\[5mm] = &
sum_{k = 0}^{infty}pars{k + 1}
bracks{{-n - 1 choose m - k}pars{-1}^{m - k}}
\[5mm] = &
pars{-1}^{m}sum_{k = 0}^{infty}pars{k + 1}pars{-1}^{k}
bracks{z^{m - k}}pars{1 + z}^{-n - 1}
\[5mm] = &
pars{-1}^{m}bracks{z^{m}}pars{1 + z}^{-n - 1}
sum_{k = 0}^{infty}pars{k + 1}pars{-z}^{k}
\[5mm] = &
pars{-1}^{m}bracks{z^{m}}pars{1 + z}^{-n - 1},
pars{-,partiald{}{z}sum_{k = 0}^{infty}pars{-z}^{k + 1}}
\[5mm] = &
pars{-1}^{m}bracks{z^{m}}pars{1 + z}^{-n - 1},
partiald{}{z}pars{z over 1 + z} =
pars{-1}^{m}bracks{z^{m}}pars{1 + z}^{-n - 3}
\[5mm] = &
pars{-1}^{m}{-n - 3 choose m} =
pars{-1}^{m}bracks{{n + 3 + m - 1choose m}pars{-1}^{m}}
\[5mm] = &
bbx{n + m + 2 choose n + 2}
end{align}





share|cite|improve this answer









$endgroup$





















    2












    $begingroup$

    The RHS can be written as $$sum_{i+j=n+m+1}binom{i}1binom{j}n$$where $binom{r}{s}:=0$ if $snotin{0,dots,r}$.



    This equals: $$binom{n+2+m}{n+2}$$
    See here for a proof of that. So RHS does not equal LHS.






    share|cite|improve this answer











    $endgroup$













      Your Answer





      StackExchange.ifUsing("editor", function () {
      return StackExchange.using("mathjaxEditing", function () {
      StackExchange.MarkdownEditor.creationCallbacks.add(function (editor, postfix) {
      StackExchange.mathjaxEditing.prepareWmdForMathJax(editor, postfix, [["$", "$"], ["\\(","\\)"]]);
      });
      });
      }, "mathjax-editing");

      StackExchange.ready(function() {
      var channelOptions = {
      tags: "".split(" "),
      id: "69"
      };
      initTagRenderer("".split(" "), "".split(" "), channelOptions);

      StackExchange.using("externalEditor", function() {
      // Have to fire editor after snippets, if snippets enabled
      if (StackExchange.settings.snippets.snippetsEnabled) {
      StackExchange.using("snippets", function() {
      createEditor();
      });
      }
      else {
      createEditor();
      }
      });

      function createEditor() {
      StackExchange.prepareEditor({
      heartbeatType: 'answer',
      autoActivateHeartbeat: false,
      convertImagesToLinks: true,
      noModals: true,
      showLowRepImageUploadWarning: true,
      reputationToPostImages: 10,
      bindNavPrevention: true,
      postfix: "",
      imageUploader: {
      brandingHtml: "Powered by u003ca class="icon-imgur-white" href="https://imgur.com/"u003eu003c/au003e",
      contentPolicyHtml: "User contributions licensed under u003ca href="https://creativecommons.org/licenses/by-sa/3.0/"u003ecc by-sa 3.0 with attribution requiredu003c/au003e u003ca href="https://stackoverflow.com/legal/content-policy"u003e(content policy)u003c/au003e",
      allowUrls: true
      },
      noCode: true, onDemand: true,
      discardSelector: ".discard-answer"
      ,immediatelyShowMarkdownHelp:true
      });


      }
      });














      draft saved

      draft discarded


















      StackExchange.ready(
      function () {
      StackExchange.openid.initPostLogin('.new-post-login', 'https%3a%2f%2fmath.stackexchange.com%2fquestions%2f3031324%2fproving-binomn1mn1-sum-k-0mk1-binomnm-kn%23new-answer', 'question_page');
      }
      );

      Post as a guest















      Required, but never shown

























      2 Answers
      2






      active

      oldest

      votes








      2 Answers
      2






      active

      oldest

      votes









      active

      oldest

      votes






      active

      oldest

      votes









      3












      $begingroup$

      $newcommand{bbx}[1]{,bbox[15px,border:1px groove navy]{displaystyle{#1}},}
      newcommand{braces}[1]{leftlbrace,{#1},rightrbrace}
      newcommand{bracks}[1]{leftlbrack,{#1},rightrbrack}
      newcommand{dd}{mathrm{d}}
      newcommand{ds}[1]{displaystyle{#1}}
      newcommand{expo}[1]{,mathrm{e}^{#1},}
      newcommand{ic}{mathrm{i}}
      newcommand{mc}[1]{mathcal{#1}}
      newcommand{mrm}[1]{mathrm{#1}}
      newcommand{pars}[1]{left(,{#1},right)}
      newcommand{partiald}[3]{frac{partial^{#1} #2}{partial #3^{#1}}}
      newcommand{root}[2]{,sqrt[#1]{,{#2},},}
      newcommand{totald}[3]{frac{mathrm{d}^{#1} #2}{mathrm{d} #3^{#1}}}
      newcommand{verts}[1]{leftvert,{#1},rightvert}$




      $ds{require{cancel}bcancel{cancel{n + 1 + m choose n + 1}} =
      sum_{k = 0}^{m}pars{k + 1}{n + m - k choose n}: {LARGE ?}}$
      .


      The right answer is
      $bbx{ds{n + m + 2 choose n + 2}}$




      $$
      bbx{mbox{Note that} {n + m - k choose n} = 0
      mbox{when} k > m}
      $$




      begin{align}
      &bbox[10px,#ffd]{sum_{k = 0}^{m}pars{k + 1}
      {n + m - k choose n}} =
      sum_{k = 0}^{infty}pars{k + 1}{n + m - k choose m - k}
      \[5mm] = &
      sum_{k = 0}^{infty}pars{k + 1}
      bracks{{-n - 1 choose m - k}pars{-1}^{m - k}}
      \[5mm] = &
      pars{-1}^{m}sum_{k = 0}^{infty}pars{k + 1}pars{-1}^{k}
      bracks{z^{m - k}}pars{1 + z}^{-n - 1}
      \[5mm] = &
      pars{-1}^{m}bracks{z^{m}}pars{1 + z}^{-n - 1}
      sum_{k = 0}^{infty}pars{k + 1}pars{-z}^{k}
      \[5mm] = &
      pars{-1}^{m}bracks{z^{m}}pars{1 + z}^{-n - 1},
      pars{-,partiald{}{z}sum_{k = 0}^{infty}pars{-z}^{k + 1}}
      \[5mm] = &
      pars{-1}^{m}bracks{z^{m}}pars{1 + z}^{-n - 1},
      partiald{}{z}pars{z over 1 + z} =
      pars{-1}^{m}bracks{z^{m}}pars{1 + z}^{-n - 3}
      \[5mm] = &
      pars{-1}^{m}{-n - 3 choose m} =
      pars{-1}^{m}bracks{{n + 3 + m - 1choose m}pars{-1}^{m}}
      \[5mm] = &
      bbx{n + m + 2 choose n + 2}
      end{align}





      share|cite|improve this answer









      $endgroup$


















        3












        $begingroup$

        $newcommand{bbx}[1]{,bbox[15px,border:1px groove navy]{displaystyle{#1}},}
        newcommand{braces}[1]{leftlbrace,{#1},rightrbrace}
        newcommand{bracks}[1]{leftlbrack,{#1},rightrbrack}
        newcommand{dd}{mathrm{d}}
        newcommand{ds}[1]{displaystyle{#1}}
        newcommand{expo}[1]{,mathrm{e}^{#1},}
        newcommand{ic}{mathrm{i}}
        newcommand{mc}[1]{mathcal{#1}}
        newcommand{mrm}[1]{mathrm{#1}}
        newcommand{pars}[1]{left(,{#1},right)}
        newcommand{partiald}[3]{frac{partial^{#1} #2}{partial #3^{#1}}}
        newcommand{root}[2]{,sqrt[#1]{,{#2},},}
        newcommand{totald}[3]{frac{mathrm{d}^{#1} #2}{mathrm{d} #3^{#1}}}
        newcommand{verts}[1]{leftvert,{#1},rightvert}$




        $ds{require{cancel}bcancel{cancel{n + 1 + m choose n + 1}} =
        sum_{k = 0}^{m}pars{k + 1}{n + m - k choose n}: {LARGE ?}}$
        .


        The right answer is
        $bbx{ds{n + m + 2 choose n + 2}}$




        $$
        bbx{mbox{Note that} {n + m - k choose n} = 0
        mbox{when} k > m}
        $$




        begin{align}
        &bbox[10px,#ffd]{sum_{k = 0}^{m}pars{k + 1}
        {n + m - k choose n}} =
        sum_{k = 0}^{infty}pars{k + 1}{n + m - k choose m - k}
        \[5mm] = &
        sum_{k = 0}^{infty}pars{k + 1}
        bracks{{-n - 1 choose m - k}pars{-1}^{m - k}}
        \[5mm] = &
        pars{-1}^{m}sum_{k = 0}^{infty}pars{k + 1}pars{-1}^{k}
        bracks{z^{m - k}}pars{1 + z}^{-n - 1}
        \[5mm] = &
        pars{-1}^{m}bracks{z^{m}}pars{1 + z}^{-n - 1}
        sum_{k = 0}^{infty}pars{k + 1}pars{-z}^{k}
        \[5mm] = &
        pars{-1}^{m}bracks{z^{m}}pars{1 + z}^{-n - 1},
        pars{-,partiald{}{z}sum_{k = 0}^{infty}pars{-z}^{k + 1}}
        \[5mm] = &
        pars{-1}^{m}bracks{z^{m}}pars{1 + z}^{-n - 1},
        partiald{}{z}pars{z over 1 + z} =
        pars{-1}^{m}bracks{z^{m}}pars{1 + z}^{-n - 3}
        \[5mm] = &
        pars{-1}^{m}{-n - 3 choose m} =
        pars{-1}^{m}bracks{{n + 3 + m - 1choose m}pars{-1}^{m}}
        \[5mm] = &
        bbx{n + m + 2 choose n + 2}
        end{align}





        share|cite|improve this answer









        $endgroup$
















          3












          3








          3





          $begingroup$

          $newcommand{bbx}[1]{,bbox[15px,border:1px groove navy]{displaystyle{#1}},}
          newcommand{braces}[1]{leftlbrace,{#1},rightrbrace}
          newcommand{bracks}[1]{leftlbrack,{#1},rightrbrack}
          newcommand{dd}{mathrm{d}}
          newcommand{ds}[1]{displaystyle{#1}}
          newcommand{expo}[1]{,mathrm{e}^{#1},}
          newcommand{ic}{mathrm{i}}
          newcommand{mc}[1]{mathcal{#1}}
          newcommand{mrm}[1]{mathrm{#1}}
          newcommand{pars}[1]{left(,{#1},right)}
          newcommand{partiald}[3]{frac{partial^{#1} #2}{partial #3^{#1}}}
          newcommand{root}[2]{,sqrt[#1]{,{#2},},}
          newcommand{totald}[3]{frac{mathrm{d}^{#1} #2}{mathrm{d} #3^{#1}}}
          newcommand{verts}[1]{leftvert,{#1},rightvert}$




          $ds{require{cancel}bcancel{cancel{n + 1 + m choose n + 1}} =
          sum_{k = 0}^{m}pars{k + 1}{n + m - k choose n}: {LARGE ?}}$
          .


          The right answer is
          $bbx{ds{n + m + 2 choose n + 2}}$




          $$
          bbx{mbox{Note that} {n + m - k choose n} = 0
          mbox{when} k > m}
          $$




          begin{align}
          &bbox[10px,#ffd]{sum_{k = 0}^{m}pars{k + 1}
          {n + m - k choose n}} =
          sum_{k = 0}^{infty}pars{k + 1}{n + m - k choose m - k}
          \[5mm] = &
          sum_{k = 0}^{infty}pars{k + 1}
          bracks{{-n - 1 choose m - k}pars{-1}^{m - k}}
          \[5mm] = &
          pars{-1}^{m}sum_{k = 0}^{infty}pars{k + 1}pars{-1}^{k}
          bracks{z^{m - k}}pars{1 + z}^{-n - 1}
          \[5mm] = &
          pars{-1}^{m}bracks{z^{m}}pars{1 + z}^{-n - 1}
          sum_{k = 0}^{infty}pars{k + 1}pars{-z}^{k}
          \[5mm] = &
          pars{-1}^{m}bracks{z^{m}}pars{1 + z}^{-n - 1},
          pars{-,partiald{}{z}sum_{k = 0}^{infty}pars{-z}^{k + 1}}
          \[5mm] = &
          pars{-1}^{m}bracks{z^{m}}pars{1 + z}^{-n - 1},
          partiald{}{z}pars{z over 1 + z} =
          pars{-1}^{m}bracks{z^{m}}pars{1 + z}^{-n - 3}
          \[5mm] = &
          pars{-1}^{m}{-n - 3 choose m} =
          pars{-1}^{m}bracks{{n + 3 + m - 1choose m}pars{-1}^{m}}
          \[5mm] = &
          bbx{n + m + 2 choose n + 2}
          end{align}





          share|cite|improve this answer









          $endgroup$



          $newcommand{bbx}[1]{,bbox[15px,border:1px groove navy]{displaystyle{#1}},}
          newcommand{braces}[1]{leftlbrace,{#1},rightrbrace}
          newcommand{bracks}[1]{leftlbrack,{#1},rightrbrack}
          newcommand{dd}{mathrm{d}}
          newcommand{ds}[1]{displaystyle{#1}}
          newcommand{expo}[1]{,mathrm{e}^{#1},}
          newcommand{ic}{mathrm{i}}
          newcommand{mc}[1]{mathcal{#1}}
          newcommand{mrm}[1]{mathrm{#1}}
          newcommand{pars}[1]{left(,{#1},right)}
          newcommand{partiald}[3]{frac{partial^{#1} #2}{partial #3^{#1}}}
          newcommand{root}[2]{,sqrt[#1]{,{#2},},}
          newcommand{totald}[3]{frac{mathrm{d}^{#1} #2}{mathrm{d} #3^{#1}}}
          newcommand{verts}[1]{leftvert,{#1},rightvert}$




          $ds{require{cancel}bcancel{cancel{n + 1 + m choose n + 1}} =
          sum_{k = 0}^{m}pars{k + 1}{n + m - k choose n}: {LARGE ?}}$
          .


          The right answer is
          $bbx{ds{n + m + 2 choose n + 2}}$




          $$
          bbx{mbox{Note that} {n + m - k choose n} = 0
          mbox{when} k > m}
          $$




          begin{align}
          &bbox[10px,#ffd]{sum_{k = 0}^{m}pars{k + 1}
          {n + m - k choose n}} =
          sum_{k = 0}^{infty}pars{k + 1}{n + m - k choose m - k}
          \[5mm] = &
          sum_{k = 0}^{infty}pars{k + 1}
          bracks{{-n - 1 choose m - k}pars{-1}^{m - k}}
          \[5mm] = &
          pars{-1}^{m}sum_{k = 0}^{infty}pars{k + 1}pars{-1}^{k}
          bracks{z^{m - k}}pars{1 + z}^{-n - 1}
          \[5mm] = &
          pars{-1}^{m}bracks{z^{m}}pars{1 + z}^{-n - 1}
          sum_{k = 0}^{infty}pars{k + 1}pars{-z}^{k}
          \[5mm] = &
          pars{-1}^{m}bracks{z^{m}}pars{1 + z}^{-n - 1},
          pars{-,partiald{}{z}sum_{k = 0}^{infty}pars{-z}^{k + 1}}
          \[5mm] = &
          pars{-1}^{m}bracks{z^{m}}pars{1 + z}^{-n - 1},
          partiald{}{z}pars{z over 1 + z} =
          pars{-1}^{m}bracks{z^{m}}pars{1 + z}^{-n - 3}
          \[5mm] = &
          pars{-1}^{m}{-n - 3 choose m} =
          pars{-1}^{m}bracks{{n + 3 + m - 1choose m}pars{-1}^{m}}
          \[5mm] = &
          bbx{n + m + 2 choose n + 2}
          end{align}






          share|cite|improve this answer












          share|cite|improve this answer



          share|cite|improve this answer










          answered Dec 8 '18 at 20:30









          Felix MarinFelix Marin

          67.9k7107142




          67.9k7107142























              2












              $begingroup$

              The RHS can be written as $$sum_{i+j=n+m+1}binom{i}1binom{j}n$$where $binom{r}{s}:=0$ if $snotin{0,dots,r}$.



              This equals: $$binom{n+2+m}{n+2}$$
              See here for a proof of that. So RHS does not equal LHS.






              share|cite|improve this answer











              $endgroup$


















                2












                $begingroup$

                The RHS can be written as $$sum_{i+j=n+m+1}binom{i}1binom{j}n$$where $binom{r}{s}:=0$ if $snotin{0,dots,r}$.



                This equals: $$binom{n+2+m}{n+2}$$
                See here for a proof of that. So RHS does not equal LHS.






                share|cite|improve this answer











                $endgroup$
















                  2












                  2








                  2





                  $begingroup$

                  The RHS can be written as $$sum_{i+j=n+m+1}binom{i}1binom{j}n$$where $binom{r}{s}:=0$ if $snotin{0,dots,r}$.



                  This equals: $$binom{n+2+m}{n+2}$$
                  See here for a proof of that. So RHS does not equal LHS.






                  share|cite|improve this answer











                  $endgroup$



                  The RHS can be written as $$sum_{i+j=n+m+1}binom{i}1binom{j}n$$where $binom{r}{s}:=0$ if $snotin{0,dots,r}$.



                  This equals: $$binom{n+2+m}{n+2}$$
                  See here for a proof of that. So RHS does not equal LHS.







                  share|cite|improve this answer














                  share|cite|improve this answer



                  share|cite|improve this answer








                  edited Dec 8 '18 at 17:01

























                  answered Dec 8 '18 at 16:53









                  drhabdrhab

                  101k544130




                  101k544130






























                      draft saved

                      draft discarded




















































                      Thanks for contributing an answer to Mathematics Stack Exchange!


                      • Please be sure to answer the question. Provide details and share your research!

                      But avoid



                      • Asking for help, clarification, or responding to other answers.

                      • Making statements based on opinion; back them up with references or personal experience.


                      Use MathJax to format equations. MathJax reference.


                      To learn more, see our tips on writing great answers.




                      draft saved


                      draft discarded














                      StackExchange.ready(
                      function () {
                      StackExchange.openid.initPostLogin('.new-post-login', 'https%3a%2f%2fmath.stackexchange.com%2fquestions%2f3031324%2fproving-binomn1mn1-sum-k-0mk1-binomnm-kn%23new-answer', 'question_page');
                      }
                      );

                      Post as a guest















                      Required, but never shown





















































                      Required, but never shown














                      Required, but never shown












                      Required, but never shown







                      Required, but never shown

































                      Required, but never shown














                      Required, but never shown












                      Required, but never shown







                      Required, but never shown







                      Popular posts from this blog

                      Le Mesnil-Réaume

                      Ida-Boy-Ed-Garten

                      web3.py web3.isConnected() returns false always